What is the solution to j + (-3) = 9?

Answers

Answer 1

Answer:

j = 12

Step-by-step explanation:

j + (-3) = 9

j = 12


Related Questions

write a verbal expression for x + y

Answers

y=Mx+-b pgigigiggihiihihohohoh

There is no options to choose from and I need to find slope help due today

Answers

♥️♥️♥️♥️♥️♥️♥️♥️♥️♥️♥️♥️♥️♥️

To find the slope we need two points.

A = ( -2 , 5 ) & B = ( 0 , 2 ) & C = ( 2 , -1 )

I choose A and B .

_________________________________

We have following equation to find the slope using two points.

[tex]slope= \frac{y(B) - y(A)}{x(B) - x(A)} \\ [/tex]

Now just need to put the coordinates in the above equation.

Let's do it...

[tex]slope = \frac{2 - 5}{0 - ( - 2)} \\ [/tex]

[tex]slope = \frac{ - 3}{2} \\ [/tex]

[tex]slope = - \frac{3}{2} \\ [/tex]

♥️♥️♥️♥️♥️♥️♥️♥️♥️♥️♥️♥️♥️♥️

I suggest to find the slope using A , C

and B , C to can understand exactly what did I do buddy.

Good luck ♥️♥️♥️♥️♥️

HELP ASAP I don’t get it

Answers

Answer:

-11/5

Step-by-step explanation:

In order to solve it u should open the mixed fraction ,i.e,

11/7÷-15/21

=11/7×-21/15

Cutting,

ans=-11/5

Mark me the brainliest

Answer:

Fraction: -11/5

Decimal form: -2.2

Mixed number form: -2 1/5

Step-by-step explanation:

Which expression is equivalent to 32 ⋅ 3−5?

Answers

Answer:

91

Step-by-step explanation:

Using the PEMDAS method (parentheses exponenets multiplication division addition subtraction), we know that since 3-5 is not in parentheses, we have to multiply before we subtract. So first, we must multiply 32 x 3 to get a product of 96. Then we must subtract, 96 - 5 = 91. Therefore, your answer is 91.

Hope this helped! Good luck!

The age of earth’s crust is 5 · 10^9 years. Express this number as an integer.

Answers

Answer:

5,000,000,000 (see below)

Step-by-step explanation:

Integers are whole numbers and their opposites. That doesn't include fractions or decimals, even if they're negative. Being negative doesn't do anything, really.

For example, 5 and –5 are integers, but 5.1 or –5.3 are not integers. They're decimals, which means they're rational.

Integers also include 0.

Whole numbers do not include 0, so they're only 1, 2, 3, 4, 5, 6, 7, 8, 9 . . . and so on.

All whole numbers are integers.

So in order to find 5 × [tex]10^{9}[/tex] as an integer, you'll have to multiply the 5 and [tex]10^{9}[/tex] together.

[tex]10^{9}[/tex] is simply nine zeros, so it's: 1,000,000,000.

5 × 1,000,000,000 = 5,000,000,000

5,000,000,000 is not a decimal or fraction—and it also isn't negative. That means it's a whole number. Which qualifies it as an integer as well.

The quotient of the difference of 5 times x cubed and 4 and x

Answers

Answer:

[tex] \frac{5x {}^{3} - 4 }{x} [/tex]

Complete the function using the function y=-2x -6
-2
0
2
4

Answers

I need more info please!!

Ruben bought 666 comic books for \$21$21dollar sign, 21. Each comic book was the same price. What was the cost for 111 comic books?

Answers

Answer:

Each comic book is $3.30

Step-by-step explanation:

$21 divided by 6= $3.50

Answer:

Each comic book is $3.30

Step-by-step explanation:

$21 divided by 6= $3.50

(y^3)^2 without exponents​

Answers

Answer:

y^6

Step-by-step explanation:

Answer:

[tex]y^{6}[/tex]

Step-by-step explanation:

Step 1:

( y³ )²       Equation

Step 2:

[tex]y^{3 * 2}[/tex]        Multiply

Answer:

[tex]y^{6}[/tex]

Hope This Helps :)

Solve the simple equation.
5(2d+2) = 3 (8d-5)

Answers

Answer:

d = 25/14

Step-by-step explanation:

5(2d+2) = 3 (8d-5)    

10d + 10 = 24d - 15   | +15

10d + 25 = 24d        | - 10d

25 = 14 d                  | :14

d = 25/14

A group of friends were working on a student film. They spent $182 on equipment, which was 26% of their total budget. What was the total budget for their student film?

I'm not sure about this question, can some please help, I will make sure to mark you brainliest! Also if you help me I give a big amount of points!

Answers

Answer:

47.32

Step-by-step explanation:

47.32 is the correct answer

Which is a factor of
25
x
2

40
x
+
16
?

A.
5
x

4
B.
5
x
+
8
C.
5
x
+
4
D.
5
x

8

Answers

Answer:

OOOOOOOOOOOOOO9ONOOOOOOOOOOOO

Find values of a and b that make both equalities true: a/b = 2/5 and 22/a = 11/6 .

Answers

Answer:

a=32b=80

Step-by-step explanation:

Find values of a and b that make both equalities true:

a/b = 2/5 and 22/a = 11/6 .

given that 22/a=11/16

cross multiply

11a=22x16

11a=352

a=352/11

a=32

put a=32 in  a/b = 2/5 to find b we have

32/b=2/5

cross multiply we have

2b=32x5

2b=160

divide both sides by 2 we have

b=160/2

b=80

Answer:

Step-by-step explanation:

First, we will start by solving: 22/a = 11/6

Solving for a

We can first cross multiply and we get this: 22 x 6 = 11a, Simplified it would be 132 = 11a. Now we know that a = 12.

Solving for b

Now knowing a = 12 we can get this. 12/b = 2/5 same thing, cross multiply. That would be: 12 x 5 = b x 2, Simplified: 60 = 2b so b = 30

7. A fish tank hold 100 gallons of water and is losing water at a rate of 1 gallons per hour. A second fish tank
contains 40 gallons of water is having 2 gallons added per hour. After how many hours will the first tank
have less water than the second? Which inequality represents the situation?
a. 100+ h > 40 - 2h
c. 40-h > 100 + 5h
b. 100-h< 40 + 2h
d. 100-h > 40 + 2h
OA
OB
ОС
D
D
Question 8
My

Answers

Answer:

B

Step-by-step explanation:

the first fish tank has 100 gallons of water and 1 gallon is removed every hour which makes 100-h

the second tank has 40 gallons in it and 2 gallons are added every hour which makes 40+2h

after 20 hours the second tank will have more gallons than the first tank

this means that 40+2h would be greater than 100-h since it's also asking you "After how many hours will the first tank  have less water than the second?"

Order these numbers from least to greatest.
8.506, 8.6, 8.5612, 8.56

Answers

Answer:

It goes 8.506, 8.56, 8.5612, 8.6

Happy to Help

Helpppppppppppppppppppppppppppppppppppp

Answers

wouldn’t that be like -5,1?

What is the value of the unknown in the equation 3m + 7 = - 23?

Answers

Answer:

Step-by-step explanation:

3m + 7 = - 23

we pass the -7 to the right with a positive sign

3m = -23 - 7

we perform the sum of negative numbers

3m = -30

we pass the -3 to the right to divide

m = -30/3

we perform the division

m = -10

2d + 7 = 9
Hellllllpppppp

Answers

Answer:

Doesn't 2+7 = 9 itself?

Step-by-step explanation:

d=3/2

first you subtract the seven then divide the 2 on both sides to get D isolated

The 7th grade team went out for lunch. Their server brought the bill that totaled $72. The team wanted to tip the server 20% and there is also a 9% sales tax. How much will the 7th grade team spend after the tip and tax are added? Round your answer to the nearest cent if necessary.

Answers

Answer:

I think it's 92.88

Step-by-step explanation:

add up 20 and 9 to get a total of 29%

then multiply .29 and 72 to get 20.88

then add 20.88 and 72 to get the total cost including tips and tax

Form a polynomial function f(x) with the given degree and zeros. You can leave your answer in factored form.
Degree 7; zeros: -7, i, 3-1, -6i

Answers

Answer:

Your search - Form a polynomial function f(x) with the given degree and zeros. You can leave your answer in factored form. Degree 7; zeros: -7, i, 3-1, -6i - did not match any image results.

Suggestions:

Make sure all words are spelled correctly.

Try different keywords.

Try more general keywords.

Try fewer keywords.

Given the following functions: f(x) = x2 – 5 and g(x) = x - 3.
Find f(g(x))

Answers

Answer:

f(g)x))= x^2 -6x +4

Step-by-step explanation:

first, plug in g(x) into f(x): f(g)x)) =(x-3)^2 -5

then, you simplify the (x-3)^2 into (x-3)(x-3): f(g)x))= (x-3)(x-3) -5

when you distribute the two (x-3) you end up with: f(g)x))= x^2 -6x +9 -5

finally, you combine like terms and subtract 5 from 9 and your final answer is : f(g)x))= x^2 -6x +4

note: x^2 is x squared or exponent of 2

If there are 9 rows of seats with 18 seats in each row there are also 6 rows of seats with 24 seats in each row how many seats are there in the auditorium

Answers

306, 9 times 18 is 162 + 6 times 24 is 144 and add 162+144 to get 306 seats in the auditorium.

Answer:

306

Step-by-step explanation:

The number of rows times the number of seats in each row is equal to the stadium chair. Using this, 9 rows of seats with 18 seats in each row is 9(18)  and 6 rows with 24 seats in each row is 6(24). Now we add 9(18) + 6(24), which is 162 + 144 which is 306 seats

NEED HELP PLEASE !

Can you conclude that the following is a rhombus, rectangle, or square?
A parallelogram's diagonals form eight congruent angles at the vertices.

Answers

B but can we see a pic

In a square, opposite sides are parallel, all sides are equal, and the angles made by the diagonals are equal, that is 45°.

What is a parallelogram?

It is a polygon with four sides. The total interior angle is 360 degrees. A parallelogram's opposite sides are parallel and equal.

In a rectangle, opposite sides are parallel and equal, but the angles made by the diagonals are different.

In a rhombus, opposite sides are parallel and all sides are equal, but the angles made by the diagonals are different.

In a square, opposite sides are parallel, all sides are equal, and the angles made by the diagonals are equal, that is 45°.

More about the parallelogram link is given below.

https://brainly.com/question/1563728

#SPJ2

Find the value of x.

Answers

Answer:

27=x

Step-by-step explanation:

Although it looks slanted, it is an 90 degree angle. Let's make an equation.

90= 55+x+8

90=   63+x

-63     -63

27=x

hope this helps!

Answer:

27

Step-by-step explanation:

180-90=90

90-55=35

35=x+8

X=27

describe the translation from the red figure to the blue figure

Answers

Answer:

Step-by-step explanation:

From the figure attached,

Coordinates of point A → (2, 3)

Coordinates of point A' → (-3, 1)

Figure shows the translation of point A to point A'.

Relation → A(2, 3) → A'(-3, 1)

                             → A'[(2 - 5), (3 - 2)]

Therefore, rule followed for the translation was,

(x, y) → [(x- 5), (y - 2)]

Quadrilateral in red has been shifted by 5 units left and 2 units down to form the image quadrilateral in blue.

Answer:

The quadrilateral in red has been shifted by 5 units left and 2 units down to form the image quadrilateral in blue.

Step-by-step explanation:

Rewrite the following expression: x^-7

Answers

1/x^7 that’s the answer. !

(6. 10)
Ka, 8)
(4.b)
(2.2)
a. What is the slope of the line? Type the answer in the box below.
The slope of the ines

Answers

Hold
up here’s the answer ima do the work then ik giveu it

Prove the polynomial identity. (2x−1)2+2(2x−1)=(2x+1)(2x−1) Drag and drop the expressions to correctly complete the proof of the polynomial identity. (2x−1)2+2(2x−1)=(2x+1)(2x−1) Response area+4x−2=(2x+1)(2x−1) Response area=(2x+1)(2x−1) Response area=(2x+1)(2x−1)

Answers

Answer:

Step-by-step explanation:

Given expression is,

(2x - 1)² + 2(2x - 1) = (2x - 1)(2x + 1)

To prove this identity we will take the left hand side of the equation and will prove equal to the right side.

(2x - 1)² + 2(2x - 1) = (2x - 1)(2x + 1)

4x² - 4x + 1 + 4x - 2 = (2x - 1)(2x + 1)

4x² - 1 = (2x - 1)(2x + 1)

(2x - 1)(2x + 1) = (2x - 1)(2x + 1) [Since a² - b² = (a - b)(a + b)]

ignore the black words on the bottom but i’ll be giving brainliest!

Answers

Answer:

2. the figure has rotatinal symetry

Step-by-step explanation:

Answer:

1. x = 4; y = 7

2. rotational symmetry.

3. No. 360°÷ 50° is not a whole number, so the points will not be evenly spaced all the way around the center.

Step-by-step explanation:

hi again ♡

if the slope of ac 6 what is the slope of bc

Answers

Answer:
Slope of BC=-1/6

Explanation:
What we know:
- the slope of AC is 6
- BC is perpendicular to AC

We know what BC is perpendicular to AC because they meet at a right angle.

To solve this question, we must understand that the slope of a second line perpendicular to the first is equal to the NEGATIVE RECIPROCAL of the slope of the first line.

For example, the slope of line A=8. The slope of the line perpendicular to line A would be -1/8.

Another example: the slope of line B=-6/7. The slope of a line perpendicular to line B would be 7/6. Remember, two negatives make a positive.

And that brings us back to line BC. Because the slope of line AC is 6, the slope of line BC would be -1/6.

I hope this helps! Please comment if you have any questions.
Other Questions
What are the domain and range for the representative exponential function in the graph below?A)Domain: y > 0Range: x RB)Domain: x RRange: y > 0C)Domain: x RRange: y 0B)Domain: y 0Range: x R How many essential amino acids must be obtained from food? a)8 b)20 c)12 d)26 What is Montags family names? 40% of 250 is equal to 60% of what number whats the degree of the polynomial 5a-2b^2+1 How are Gymnosperm and Angiosperm plants different? CAN SOMEONE HELP PLEASE!!!!Select the sentence that has the correct punctuation.A. His book, a soggy mass of paper, was barely recognizable.B. His book, a soggy, mass of paper was barely recognizable. The origin of a muscle is generally:.The stable and proximal attachment.B. The stable and distal attachment.O C.The moveable and distal attachment.D.The moveable and proximal attachment. imagine that you are plotting points on a coordinate plane. each axes is labeled with integers from -5 to 5. explain how you would plot a point at the location (-2 1/2, -4) write three or four sentences The best clues to use to make inferences are: A. details and background knowledge. B. questions and answers. C. introductory paragraphs and numbered steps. D. descriptions of expectations. E. explicitly stated codes of conduct. What is a row of data in a database called?FieldFileRecordTitle Help ASAP). Which relation is a function. Where the hand is at thats graph A across is graph B. The graph under graph a is ( graph C). Under graph B is (graph D). Need answer now please and thank you. [ I chose B but not 100% sure] How to get passed administrattor on macbook air to get games and stuff. Find the value of x: Write an equation for the problem. Use n for the variable. *$5 less than the amount Steven has is $25. The mean value of land and buildings per acre from a sample of farms is $1400, with a standard deviation of $200. The data set has a bell-shaped distribution. Assume the number of farms in the sample is 74. (a) Use the empirical rule to estimate the number of farms whose land and building values per acre are between $1200 and $1600. (b) If 29 additional farms were sampled, about how many of these additional farms would you expect to have land and building values between $1200 per acre and $1600 per acre? please i rlly need halp 45 degrees+2k+k=180can you help me with this? The movement of individuals into an area is calledA. DemographyB. Carrying capacityC. ImmigrationD. Emigration 8. Which city do millions of Muslims go to each year to perform specific rituals?